• Profile
Close

Do You Think These Answers are Heart to Miss? 10 MCQ challenge

M3 India Newsdesk Sep 29, 2022

The theme of this year's World Heart day is 'USE HEART FOR EVERY HEART'. Let's learn more about the diagnosis of heart diseases. Are you up for the challenge?


 

Cardiology quiz

  1. A 70-year-old man with a history of CAD and MI presents with progressive dyspnea and fatigue. A CTA of the chest is performed. Which of the following is the most likely diagnosis?


  1. A 42-year-old female is presented with chest pain after running a marathon. She has h/o hypertension and is on amlodipine. O/E: BP: 190/110 mmHg, PR: 90bpm. A bilateral carotid bruit is noted. CVS examination is normal. ECG shows ST depression V1-V3. CAG demonstrates focal stenosis in the left circumflex artery. What is the next step in management?

  1. A 65-year-old woman, K/c/o Cushing syndrome of unclear aetiology with biochemical evidence of hypercortisolemia for over a decade. H/o osteoporosis that is treated with alendronic acid. H/o hypertension, dyslipidemia, obesity and T2DM managed with metformin and liraglutide. She has h/o deep venous thrombosis and was recently admitted for community-acquired pneumonia. What is the most significant concern for mortality risk in this patient?

  1. A 35-year-old male is scheduled to undergo knee surgery. His pre-operative ECG was done and image is attached below. No c/o palpitations. What is the next step in the management?


  1. A 65-year-old female presented with ℅ dyspnea since 3 months. On examination basal crackles were heard. CVS exam S1 S2 S3+. She had past history of dysphonia and digestive problems. Echo showed EF 40%. CMR was done shown below. What is the most likely diagnosis?

  1. Which of the following risk factors has the weakest association with sudden cardiac death in patients with HCM?

  1. A 64-year-old man presents with transient ischaemic occlusion. Aortogram shows proximal right internal carotid artery 90% stenosis. He is planning for carotid stenting. Which is the best catheter to cannulate the innominate artery?


  1. Renal denervation therapy is contraindicated in all EXCEPT___.

  1. A 38-year-old female presents with severe chest pain, ECG showing ST elevation. IVUS of LAD was performed. What is the diagnosis?


  1. Abnormal ankle-brachial pressure index is associated with a high risk for cardiovascular toxicity with which chemotherapy drug?
Only Doctors with an M3 India account can read this article. Sign up for free or login with your existing account.
4 reasons why Doctors love M3 India
  • Exclusive Write-ups & Webinars by KOLs

  • Nonloggedininfinity icon
    Daily Quiz by specialty
  • Nonloggedinlock icon
    Paid Market Research Surveys
  • Case discussions, News & Journals' summaries
Sign-up / Log In
x
M3 app logo
Choose easy access to M3 India from your mobile!


M3 instruc arrow
Add M3 India to your Home screen
Tap  Chrome menu  and select "Add to Home screen" to pin the M3 India App to your Home screen
Okay